Wirkungsgrad des Stirlingmotors und Satz von Carnot

Ich möchte die Effizienz dieses Stirling-Zyklus für ein ideales Gas berechnen p v = n R T

aus Nolting übernommen.  Grundkurs Theoretische Physik 4. Spezielle Relativitätstheorie und Thermodynamik

Die mechanische Arbeit ist

Δ W 12 = v 1 v 2 p ( v ) d v = n R T 2 ln v 2 v 1 Δ W 23 = Δ W 41 = 0 Δ W 34 = n R T 1 ln v 1 v 2
Auf den Isothermenkurven die Veränderung der inneren Energie Δ U = Δ W + Δ Q ist Null.
Δ Q 12 = Δ W 12 > 0 Δ Q 34 = Δ W 34 < 0
Auf den isochoren (isovolumetrischen) Kurven stehen die Wärmemengen
Δ Q 23 = C v ( T 1 T 2 ) < 0 Δ Q 41 = C v ( T 2 T 1 ) > 0
Der Wirkungsgrad ist dann
η = Δ W Δ Q
Δ Q ist die zugeführte Wärme, also die Summe aller Wärmemengen > 0 :
Δ Q = Q 12 + Q 41 = n R T 2 ln v 2 v 1 + C v ( T 2 + T 1 )
Δ W ist die gesamte mechanische Arbeit:
Δ W = W 12 + Δ W 34 = n R ( T 2 T 1 ) ln v 2 v 1

So ist schließlich die Effizienz

η = T 2 T 1 T 2 + C v ( T 2 T 1 ) n R ln v 2 / v 1 < η C .
Er ist kleiner als der Wirkungsgrad des Carnot-Zyklus. Es sollte aber gleich sein, wenn alle Prozesse reversibel ablaufen.

Die Berechnungen sind einem Lehrbuch (Nolting: Grundkurs Theoretische Physik 4) entnommen, das den Leser tatsächlich als Frage auf dieses Problem hinweist. Meine einzige Erklärung ist, dass dieser Prozess nicht umkehrbar ist, aber ich weiß nicht, wie ich das sagen soll, ohne tatsächlich zu sehen, wie die isothermen und isochoren Prozesse realisiert werden.

Also meine Fragen sind:

  • Ist dies ein Widerspruch zum Satz von Carnot, dass die Effizienz η C = 1 T 1 / T 2 ist das gleiche für alle reversiblen Wärmekraftmaschinen zwischen zwei Wärmebädern?
  • Ist dieser Kreislauf reversibel?
  • Kann man nur mit einer Zahl wie der obigen sagen, ob ein Prozess reversibel oder irreversibel ist?
Du verschätzt den Wirkungsgrad. Für einen beliebigen Motorzyklus haben Sie Q H = Q L + W , und e = W Q H . Sie teilen nicht durch Ihre Eingangswärme.
Ich habe die Frage bearbeitet, um zu verdeutlichen, was ich berechne. Der Nenner im Wirkungsgrad sollte die Wärme sein, die in den Motor eingebracht wird. So wie ich die Wärme berechne, ist es das Richtige > 0 . Sind Sie einverstanden?
Bezogen auf die Frage der Reversibilität von Kurven im thermodynamischen Zustandsraum: physical.stackexchange.com/questions/78405/…

Antworten (5)

Überarbeitete Antwort. 01.07.2017

Das ist kein Widerspruch, denn Ihre Analyse beinhaltet nur, was mit dem gasförmigen Arbeitsstoff im Stirlingmotor passiert, und vernachlässigt eine entscheidende Komponente des Motors, den Regenerator. Wenn der Regenerator bei der Effizienzanalyse nicht als Bestandteil des Motors einbezogen wird, haben wir kein Gerät, das sich als Wärmekraftmaschine zwischen zwei Temperaturen qualifiziert, und wir sollten daher nicht erwarten, dass es sich an Carnot hält Satz, wie ich in der Originalversion dieser Antwort angegeben habe.

Wenn wir jedoch den Regenerator richtig berücksichtigen, dann stellen wir fest, dass der Wirkungsgrad des Motors der Carnot-Wirkungsgrad ist .

Natürlich ist die ganze Analyse hier eine idealisierte, bei der wir zum Beispiel davon ausgehen, dass es keine Energieverluste durch Reibung in den Komponenten des Motors gibt.

Einzelheiten.

Ein Stirlingmotor ist komplexer als der P - v Das in der Fragestellung gezeichnete Diagramm scheint darauf hinzudeuten. Wenn wir den Motor konzeptionell auf seine einfachste Form reduzieren, enthält er zwei grundlegende Komponenten:

  1. Ein gasförmiger Arbeitsstoff . Dies ist der Teil des Motors, dessen thermodynamischer Zustand entlang der Kurve in der wandert P - v Diagramm.
  2. Ein Regenerator . Dieser Teil des Motors absorbiert und speichert die Energie, die der gasförmige Arbeitsstoff während des Prozesses durch Wärmeübertragung abgibt 2 3 und gibt diese Energie dann während des Prozesses an den gasförmigen Arbeitsstoff zurück 4 1 .

Der entscheidende Punkt ist, dass bei Einbeziehung des Regenerators während der Prozesse keine Nettowärmeübertragung in den oder aus dem Motor erfolgt 2 3 und 4 1 . Die Energie, die das gasförmige Arbeitsmittel während des Prozesses verlässt 2 3 durch Wärmeübertragung wird im Regenerator gespeichert, und diese Wärme wird dann während des Prozesses wieder an das Arbeitsmittel abgegeben 4 1 . Während dieser Phasen des Zyklus wird keine Wärme zwischen dem Motor und seiner Umgebung übertragen.

Daraus folgt, dass die einzige auf den Motor als Ganzes übertragene Wärme während übertragen wird 1 2 . Dies qualifiziert das Gerät als Wärmekraftmaschine (siehe alte Antwort unten) und der Wirkungsgrad des Motors wird dann als Verhältnis der Nettoarbeitsleistung dividiert durch die Wärmezufuhr im Prozess berechnet 1 2 . Dies ergibt den Carnot-Wirkungsgrad , wie er sollte.

Meine ursprüngliche Antwort behauptete, dass der gezeichnete Zyklus nicht den Betrieb einer Wärmekraftmaschine darstellt, die zwischen zwei Temperaturen arbeitet, aber ich habe den Regenerator vernachlässigt, und ich glaube, das haben Sie implizit auch in der ursprünglich durchgeführten Berechnung getan, und dies ergab der falsche Wirkungsgrad.

Ursprüngliche, unvollständige Antwort.

Es gibt keinen Widerspruch. Der Stirling-Zyklus, den Sie oben gezeichnet haben, ist reversibel, arbeitet jedoch nicht zwischen zwei Reservoirs bei festen Temperaturen T 1 und T 2 . Die isovolumetrischen Teile des Zyklus arbeiten bei sich ständig ändernden Temperaturen (denken Sie an das ideale Gasgesetz).

Alter Nachtrag. Beachten Sie, dass in der Thermodynamik eine Wärmekraftmaschine zwischen (zwei Reservoirs bei) Temperaturen arbeiten (oder arbeiten) soll T 1 und T 2 vorausgesetzt, die gesamte Wärme, die es aufnimmt oder abgibt, erfolgt bei einer dieser beiden Temperaturen.

Um dieser Definition Glaubwürdigkeit zu verleihen (die im Wesentlichen in den meisten Diskussionen über Wärmekraftmaschinen enthalten ist, die ich gesehen habe), hier ein Zitat aus Fermis Thermodynamik-Text:

Im vorangegangenen Abschnitt haben wir einen reversiblen zyklischen Motor beschrieben, den Carnot-Motor, der eine Menge Arbeit verrichtet L während jedes seiner Zyklen durch Aufnahme einer Wärmemenge Q 2 aus einer Quelle bei Temperatur t 2 und Abgabe einer Wärmemenge Q 1 zu einer Quelle bei der niedrigeren Temperatur t 1 . Wir werden sagen, dass ein solcher Motor zwischen den Temperaturen arbeitet t 1 und t 2 .

Dann glaube ich, ich habe nicht verstanden, was das Arbeiten zwischen zwei Reservoirs bei fester Temperatur eigentlich bedeutet. Der Carnot-Zyklus besteht aus Adiabaten und Isothermen. Temperatur ändert sich, aber es wird keine Wärme übertragen. "Betrieb" bedeutet also im Grunde Wärmeübertragung? Sie haben auch gesagt, dass der Zyklus umkehrbar ist. Sind alle Kurven, die innerhalb der gezogen werden können p - v -Ebene reversibel?
@frankundfrei Ja; "Betrieb zwischen zwei Temperaturen" bedeutet in diesem Zusammenhang, dass alle Wärmeübertragungen, die während des Zyklus auftreten, bei der einen oder anderen Temperatur stattfinden. Was die Frage der Reversibilität betrifft, bin ich mir nicht sicher, ob es angebracht ist, eine Kurve im thermodynamischen Zustandsraum selbst als reversibel oder irreversibel zu bezeichnen. Ich denke, wir müssen eine differenziertere Frage beantworten wie: Wenn wir einen idealisierten physikalischen Prozess durchführen und seine aufeinanderfolgenden Zustände durch eine kontinuierliche Kurve im Zustandsraum gut angenähert werden können, kann dieser Prozess dann umgekehrt durchgeführt werden?
Könnten Sie diese Definition von "Operation" in Ihre Antwort aufnehmen? Weder in der Vorlesung, die ich besucht habe, noch in den von mir verwendeten Lehrbüchern bin ich auf eine genaue Definition gestoßen, daher könnte sie auch für andere hilfreich sein. Danke für Ihre Hilfe!
@frankundfrei Klar. Ich habe eine Bearbeitung vorgenommen.
@joshphysics: Der Beweis des Satzes von Carnot, dh ein reversibler Zyklus hat die gleiche Effizienz wie ein Carnot-Zyklus, geht nicht davon aus, dass die Wärme isotherm auf das Gas übertragen wird, daher verstehe ich nicht, warum die isovolumetrische Übertragung von Wärme etwas ändert. Wenn das Arbeitsmedium in thermischen Kontakt mit einer heißen Quelle kommt, können wir das Fluid bei konstantem Volumen erhitzen, bis es die Temperatur der heißen Quelle erreicht. Wir brauchen nicht, dass sich die Temperatur der heißen Quelle ändert, oder?
@JoshuaBenabou Es ist lange her, dass ich über dieses Zeug nachgedacht habe, aber ich bin skeptisch gegenüber Ihrer Behauptung über die Allgemeingültigkeit von Carnots Theorem. Soweit ich das beurteilen kann, vergleicht das Theorem normalerweise alle Wärmekraftmaschinen, die zwischen festen Reservoirs mit bestimmten Temperaturen betrieben werden: en.wikipedia.org/wiki/Carnot%27s_theorem_(thermodynamics)
@joshphysics: In der Tat spricht das Theorem von Wärmekraftmaschinen, die zwischen zwei festen Reservoirs mit festen Temperaturen arbeiten. Ich persönlich denke, dass Ihre Antwort nicht ganz richtig ist, da das OP die Effizienz des Stirling-Zyklus unter der Annahme berechnet hat, dass er nur zwischen zwei Reservoirs mit festen Temperaturen arbeitet. Es ist daher nicht reversibel (da die Wärmeübertragung über eine nicht unendlich kleine Temperaturdifferenz Entropie erzeugt). Daher ist es sinnvoll, dass der Wirkungsgrad des von ihm gefundenen Stirling-Kreisprozesses geringer ist als der eines Carnot-Kreisprozesses, der zwischen denselben beiden Temperaturen betrieben wird.
@JoshuaBenabou Ich habe dank Ihrer Kommentare ziemlich viel darüber nachgedacht und meine Antwort entsprechend bearbeitet. Insbesondere glaube ich nicht, dass ein idealer Stirlingmotor irreversibel ist, wie Sie angeben. Die Wärmeübertragung zwischen dem Arbeitsstoff des Motors und dem Regenerator ist idealerweise reversibel. Darüber hinaus wird bei entsprechender Analyse des Motors (siehe überarbeitete Antwort) der Carnot-Wirkungsgrad wie gewünscht erhalten. Im Allgemeinen hatten Sie jedoch Recht mit Ihrer Skepsis – die ursprüngliche Antwort war äußerst unvollständig und irreführend.

Der Stirling-Zyklus, wie Sie ihn beschreiben, ist nicht umkehrbar. Die Übertragung von Wärme aus Wärmereservoirs entlang der Pfade 4->1 und 2->3 ist kein reversibler Prozess, da Wärme zwischen zwei Objekten mit unterschiedlichen Temperaturen übertragen wird. Um den Prozess umzukehren, müssten Sie spontan Wärme von einem kälteren zu einem heißeren Reservoir übertragen, was gegen den 2. Hauptsatz der Thermodynamik verstößt.

Stirlingmotoren werden oft als reversibel bezeichnet, dies erfordert jedoch ein spezielles Verfahren. Beachten Sie, dass die Wärme, die entlang 4->1 in den Motor übertragen wird, die gleiche ist wie die Wärme, die entlang 2->3 aus dem Motor übertragen wird, und dass 4->1 und 2->3 zwischen denselben zwei Temperaturen arbeiten. Daher kann ein Carnot-effizienter Stirlingmotor konstruiert werden, wenn die Wärme innerhalb des Motors entlang dieser Wege isotherm übertragen wird. Dies wird mit einem "Regenerator" erreicht, einer thermischen Masse, die die in 2->3 freigesetzte Energie speichert und sie auf dem Weg 4->1 an das Gas zurückgibt. Sie können sehen, dass der Regenerator zwischen T2 und T1 kontinuierlich in der Temperatur variieren und beim Durchgang Wärme isotherm mit dem Gas austauschen muss.

Beachten Sie, dass alle reversiblen Motoren mit der gleichen Effizienz arbeiten müssen. Dies folgt aus den Definitionen von Effizienz und Entropie. Ein reversibler Motor arbeitet mit 0 Entropieänderung. Δ S = Q h T h + Q c T c , Also Δ S = 0 impliziert Q h T h = Q c T c oder Effizienz = Q h Q c Q h = T h T c T h

Ich verstehe den "Regenerator" nicht ganz, aber ich denke, Ihr Argument zur Irreversibilität der Pfade 4-> 1 und 2-> 3 ist auch nicht korrekt Δ S = 0 im Kreislauf
@richard - Sie müssen die Entropieänderung der gesamten Welt berücksichtigen, nicht nur des Arbeitsmediums; jeder geschlossene Kreislauf hat eine Entropieänderung von 0 für das Arbeitsfluid; Nur reversible Motoren haben eine Entropieänderung von 0 für das Arbeitsmedium und die thermischen Reservoirs.
Ja, das sage ich, wenn ein Teil des Zyklus irreversibel wäre, wäre die Entropieänderung des Arbeitsfluids und auch der gesamten Welt ungleich Null.
@ Richard. für ein ideales Gas mit fester Teilchenzahl hat jeder Punkt im P,V-Diagramm eine bestimmte Entropie (diese lässt sich zB aus der Sackur-Tetroden-Gleichung berechnen). Daher hat jeder geschlossene Kreislauf eine 0-Änderung der Entropie des Arbeitsmediums. Behaupten Sie, dass alle geschlossenen Kreisläufe reversibel sind? Auch bestimmte reversible Prozesse, wie die isotherme Expansion, verändern die Entropie des Arbeitsmediums. Sie müssen die gesamte Entropieänderung des Arbeitsmediums und der Reservoirs zusammen betrachten.
Nein, ich behaupte nicht, dass jeder geschlossene Kreislauf reversibel ist. Ich sage, der Zyklus im obigen Bild ist umkehrbar. Alle Teile sind reversibel, einschließlich 4-1 und 2-3.
@richard: Auf dem Weg 4-1 nimmt die Entropie des Gases zu. Wenn die Wärme aus einem Reservoir mit der Temperatur T1 strömt, nimmt die Entropie des Reservoirs ab, aber sie nimmt weniger ab, als die Entropie des Gases zunimmt. Die Nettoänderung der Entropie ist positiv, und der Prozess ist irreversibel. Die einzige Möglichkeit, 4-1 reversibel zu machen, besteht darin, Wärme aus einer Reihe von Reservoirs bei jeder Zwischentemperatur zwischen T4 und T1 isotherm zu übertragen. Das ist die Funktion des Regenerators, die Sie nicht ganz verstehen.
Ich denke, die Pfade 4-1 und 2-3 im obigen Bild sind die gleichen, wie Sie erklärt haben: Satz von Reservoirs bei mittleren Temperaturen.
@richard: Es ist eine Frage, woher die Hitze in der 4-1-Phase kommt und wohin sie in der 2-3-Phase geht. In einem idealen Stirlingmotor wird die Wärme intern zum Regenerator in 2-3 und zurück vom Regenerator in 4-1 übertragen, ohne Verlust oder Gewinn nach oder von außen. Daher eine Nulländerung der Entropie in diesen Stadien. Echte Stirlingmotoren tun dies nicht zu 100% effizient, aber der ideale Stirlingmotor hat in diesen Phasen eine Entropieänderung von Null. Das PV-Diagramm ist in beiden Fällen dasselbe, aber die Entropie-gegen-T-Diagramme wären unterschiedlich.

In einem idealen Stirling-Zyklus haben die isochoren Schritte einen Wärmeaustausch über eine infinitesimale Temperaturdifferenz, die dadurch aufrechterhalten wird, dass der Regenerator einen kontinuierlichen Temperaturgradienten zwischen den heißen und kalten Reservoirs aufweist. Das Gas kann dann entsprechend diesem Gradienten abkühlen oder erwärmen. Dies ist der sehr ideale Teil des Designs, der eine Nulländerung der Entropie während der beiden isochoren Stufen ermöglicht. Diese Wärme wird nur intern hin- und hergeschoben, sodass der einzige tatsächliche Austausch mit der Außenwelt über das heiße Reservoir nach innen und über die Kälte nach außen erfolgt. Daher der ideale Wirkungsgrad. Ich bin mir nicht sicher, ob es richtig ist, das, was in den Regeneratorstufen passiert, als isotherm zu bezeichnen. Die Temperatur ändert sich kontinuierlich, aber im Idealfall immer über einen verschwindend kleinen Unterschied. Gibt es dafür einen allgemein gebräuchlichen Begriff? Dennoch,

Ich habe bei meinen Recherchen im Internet zum Thema Stirlingmotoren festgestellt, dass viele Quellen diese Ideen verwirren. Ich habe oft Effizienzanalysen gesehen, die die Wirkung des Regenerators völlig außer Acht lassen. Dies hängt möglicherweise damit zusammen, dass isochore Prozesse normalerweise nicht mit einer Nulländerung der Entropie verbunden sind, aber im Fall des Stirlingmotors handelt es sich um eine ganz besondere Art dieses Prozesses, indem ein Regenerator verwendet wird.

Der ideale Stirlingmotor hat den gleichen Wirkungsgrad wie der Carnot-Zyklus, aber sein Vorteil besteht darin, dass er den Bau echter Motoren ermöglicht, die zwar möglicherweise keine perfekten isothermen und völlig glatten isochoren Stufen des Regenerators erreichen, aber nahe kommen und sind viel praktikabler als die Möglichkeit, einen praktischen Carnot-Motor zu bauen.

In Wirklichkeit erreichen real hergestellte Stirlingmotoren nicht den vollen idealen Carnot-Wirkungsgrad, aber viele sind viel besser als andere Arten von Wärmekraftmaschinen.

Abschließend unter Berücksichtigung des idealen Stirlingmotors:

(1) Der ideale maximale Wirkungsgrad des Carnot-Motors wird erreicht. (2) Ihre Rechnung widerspricht dem nicht, weil sie falsch ist. Sie rechnen die in den isochoren Phasen ausgetauschte Wärme in die Kosten ein, während die einzigen Kosten die externe Wärmezufuhr während des isothermen Arbeitstakts sind. (3) Dieser Zyklus ist reversibel, da es während der isochoren Phasen keine Entropieänderung gibt. (4) Das Diagramm allein reicht nicht aus, um dies zu zeigen, da wir auch wissen müssen, dass der ideale Regenerator den dritten Punkt ermöglicht. Das heißt, wenn Sie den Regenerator entfernen, ist das Diagramm immer noch dasselbe.

Das Problem ergibt sich aus der 9. Gleichung. Beachten Sie, dass sich die Wärme, die während der beiden Prozesse 4-1 und 2-3 übertragen wird, gegenseitig aufhebt. Die Wärme Q41 wird an den Regenerator abgegeben und dann von ihm durch das Arbeitsmaterial des Systems wieder absorbiert. Diese Wärmemenge wird "nicht" von dem heißen Reservoir an das System abgegeben oder von dem kühlen Reservoir absorbiert, sondern wird gewissermaßen "reversibel" zwischen zwei Teilen des Arbeitsmaterials selbst übertragen. Also, einschließlich der Hitze Q41 in der 9. Gl. als Teil der Wärme, die vom Heißbewahrer auf das System "übertragen" wird, ist der falsche Punkt in den obigen Berechnungen, der zu dem falschen Ergebnis in der 11. Gleichung geführt hat.

Willkommen bei Physics.SE! Ich schlage Folgendes vor: 1) Machen Sie die Tour ! 2) Wenn Sie gute Fragen und Antworten sehen, stimmen Sie für sie ab, indem Sie auf die grauen Dreiecke klicken , da die Glaubwürdigkeit des Systems auf dem Ruf basiert, den Benutzer gewinnen, wenn sie ihr Wissen teilen. 3) Wenn Sie eine gute Frage haben, stellen Sie sie! Denken Sie daran, wenn Sie dies tun, und erhalten Sie eine zufriedenstellende Antwort, um sie zu akzeptieren, indem Sie auf das grüne Häkchen klicken. Wenn Sie eine Frage haben, sehen Sie auch in den Richtlinien der Hilfe nach .

Jeder Zyklus auf dem pv-Diagramm ist reversibel. Wenn Sie nach Q auflösen, müssen Sie integrieren, und der Integrationsprozess selbst beinhaltet dT, was bedeutet, dass der Temperaturunterschied unendlich klein ist, wodurch der Prozess reversibel wird. Die von Ihnen abgeleitete Formel für den Wirkungsgrad des Stirlingmotors ist korrekt, außer dass die Anzahl der Mole (n) sich hätte aufheben müssen. Der Wirkungsgrad des Stirlingmotors ist geringer als bei Carnot und das ist in Ordnung. Wie einer von Ihnen erwähnt hat, kann man es nicht mit Carnot vergleichen, da der Wärmeaustausch im Carnot-Zyklus bei zwei festen Temperaturen stattfindet, während im Stirlingmotor der Wärmeaustausch auch entlang der beiden Prozesse mit konstantem Volumen stattfindet, bei denen die Temperaturen variieren. Im Carnot-Zyklus gibt es keinen Wärmeaustausch entlang der adiabatischen Kurven, entlang derer sich die Temperatur ändert. Ich hoffe das hilft.